Download as pdf or txt
Download as pdf or txt
You are on page 1of 59

‫بسم هللا الرحمن الرحيم‬

, ‫ حمدي‬, ‫ نايل‬, ‫مرعي وصحصاح‬/‫هذه الملزمة شاملة لجميع أسئلة مذكرة ( د‬


)‫موقع طبيعي‬
‫نرجوا منكم عدم ارسالها الي اي احد حتي تكتمل وحتي يتم التأكد من جميع‬
‫ بشكل نهائي‬222 ‫االسئله وتم التأكد فقط حتي سؤال رقم‬
‫مالذ‬/ ‫احمد جابر و د‬/ ‫مع تحياتي د‬

ORTHOPEDIC QUESTIONS

1/ Patient suffering from morning stiffness this is a sign of :-


a- swelling of the body tissues
b- prolonged stiffness with inflammation.
c- muscle inflammation.
d- bonny projection at the joint

2/ Patient is suffering from morning stiffness; the most likely developing condition is due:-
1-Bones are growing beyond their margins
2- Fasciitis 1(refers to an inflammation of the fascia )
3- Accumulation of lactic acid
4- Muscle Spasm

3/ A 14-year old girl place excessive valgus stress to the right elbow during a fall from a
bicycle. Her forearm was in supination at the moment the valgus stress was applied. Which
of the following is most likely involved in this type of injury?
a-Ulnar nerve
*cubital valgus fracture means move the
b-Extensor carpi radialis
forearm away from the humreus which
c-Brachioradialis causes fructure of medial epicondyle Fracture
d-Annular ligament .causes ulnar nerve injury.

4/ joint play ROM which occurs not named with:-


a-necessary to completion of joints ROM
b-Passively occur
c-under voluntary control

5/ Sterno clavicular in forced by except:-


A-sterno clavicular lig
B-intra articular disc
C-intra calvicular lig
D-acroimo calviclar lig

6/ shoulder dislocation head humerus fallen:-


A- posterior
B- inferior
C- anterior

7/ Volkmans ischemic disease Most dependable signe for this pt is:-


a-Pain
b-cynasoed color of hand
c-weak ext
d-weak flex

8/ What not true about curvature of cervical region:-


A-cervical curvature lordosis increases preesure on disc
B- cervical curvature lordosis increases pressure on lig
C- cervical curvature lordosis increases pressure on facet jiont
D-Lumber and throracic disorders not accompiened with cervical curvature

-: ‫معلومه مهمه وهدية من د مرعي‬


* supracondylar --> median nerve injury
* mid humerus --> radial nerve injury
* upper humerus --> brachial nerve injury
* The cause of cubital valgus fracture of --> lateral condyle Fracture
*The cause of cubital varus fracture of --> supracondylar Fracture

9/ what is not correct about bone:-


a-contain 90% of inorganic tissues
b-compose of 2 types sponge ,compact
c-bone is hard
d-has blood supply,nerve endings

10/ Ankylosing means :


a- loosed packed position
Ankylosed=close pack position
b- joint stiffness
c – closed packed position

11/ Pt complain of groin pain with limited abduction with external rotation you expect the
diagnosis is:-
a-inguinal hernia
b- adductor strain
c- abductor sprain
d- external obliques weakness

12/ In clinical examination you must do all of the following except:-


a-observe the patient examine both affected unaffected
b-review patient sheet
c-focus only on areas of patient`s complains
d-respect the sesitizers of patients

13/ fixed stiffness of 1st IPJ joint of big toe called:-


a-hallucis varus
b-hallucis valgus
c-hammer toe
d-hallucis rigidus

14/ Which nerve supplies the muscle in the picture?


a. Axillary
b. Supascapular
c. Long thoracic
d. Musculocutaneous

15/ A patient presents to an outpatient physical therapy clinic with a severed ulnar nerve in
the upper extremity. What muscle is still active and largely responsible for the obvious
hyperextension at the metacarpophalangeal (MCP) joints of the involved hand?
A. Dorsal interossei
B. Volar interossei ‫ده كتاب المسل تست عشان السؤال‬183
C. Extensor carpi radialis brevis
D. Extensor digitorum

16/ knee joint ROM:-


a- flexion 110 – extension 0
b- flexion 130 – extension 0
c- flexion 140 – extension 0

17/ This angle is:


a) k angle
b) Q angle
c) C angle

18/ Observing a patient in a standing position, the therapist notes that an angulations
deformity of the right knee causes it to be located medially in relation to the left hip and
foot. This condition is commonly referred to as:
a-genu varum
b-genu valgum
c-pes cavus
d-none of the above.

19/Inversion, plantarflexion,adduction A 10-year-old boy presents to outpatient physical


therapy with complaints of diffuse pain in the right hip, thigh, and knee joint. The patient
was involved in a motor vehicle accident 3 weeks ago. He is also obese and has significant
atrophy in the right quadriceps. The right lower extremity is held by the patient in the
position of flexion, abduction and lateral rotation. Which of the following is most likely the
source of the patient’s signs and symptoms?
a-Greater trochanteric bursitis
19/C:-slipped femoral capital epiphysis is most common in the adolescent
b-Avascular necrosis period (boys aged 10-16 y, girls aged 12-14 y). Males have 2.4 times the
c-Slipped femoral capital epiphysis risk compared with females, Obesity is a risk factor because it places more
d-Septic arthritis , shear forces around the proximal growth plate in the hip at risk
Prodromal symptoms (eg, hip or knee pain, limp, decreased range of
motion) , Patients often hold their affected hip in passive external
rotation. , gait pattern Antalgic – Shortened stance phase on the affected
side ,Out-toeing.

20/ Normal ROM of hip joint :


a- extension 35 --- flexion 125
b- extension 10 ---- flexion 125
c- extension 0 -----flexion 125

21/ Patient with disc herniation and decrease back


motion P.T ASSIST:
A- sternocostal angle
b-lumbosacral angle
c-cobbs angle
d-cubitus angle

22/ While assessing the standing posture of a patient, the therapist notes that a spinous
process in the thoracic region is shifted laterally. The therapist estimates that T2 is the
involved vertebra because he or she notes that it is at the approximate level of the:
a- Inferior angle of the scapula
b- Superior angle of the scapula
c- Spine of the scapula
d- Xiphoid process of the sternum

-:‫معلومة ع الماشي‬
Thoracic outlet syndrome (TOS) is a syndrome involving compression at the superior thoracic outlet
[1]
involving compression of a neurovascular bundle passing between the anterior scalene and middle
scalene.[2] It can affect the brachial plexus (nerves that pass into the arms from the neck), and/or the
subclavian artery or vein (blood vessels that pass between the chest and upper extremity).
23/ In thoracic outlet syndrome is:
a- superior to operative thoracic
b- posterior to operative thoracic
c- inferior to operative thoracic
d- anterior to operative thoracic

24/The thoracic outlet syndrome, is due to :


a- Upper opening chest
b- Lower opening chest
c- Lateral opening chest
d- Anterior opening chest

25/Pt walk with extended back during bearig on leg & difficulty go up ramp this your plain is
to:-
A- stretching hip ex
25/a-mean stretching hip exercises, if there is no this choice we
B- Strengthening hip ex choose strengthening hip extensor, because stretch come before
C- Strengthening knee flexor strength.
D- Stretching knee flexor

26/ which muscle make hip abduction ;flexion ;internal rotation :


a- gluteus medius
b- external oliques
c- sartorious
d- tensor fascia latae

27/ which muscle make shoulder ;flexion ;external rotation”:


a- subscapularis
b- teres major
c- teres minor
d- pectoralis major

28/ during ex there is subtalar pronation that’s mean:-


a-femur,tibia,pelvic internal rotation
b-femur,tibia,pelvic ext rot 28/ A and D the same
C-femur ,tibia int rot with pelvic ext rot
d-femur ,tibia internal rot with pelvic int rot

29/ Kumar, a 31 years old motorcyclist sustained injury over his righthip joint. X-ray revealed
a posterior dislocation of the right hip joint.The clinical attitude of the affected lower limb
will be:
A. External rotation, extension & abduction
B. Internal rotation, flexion & adduction
C. Internal rotation, extension & abduction
D. External rotation, flexion & abduction

30/ which muscle act in Med rot,adduction and extension of shoulder joint:
a- pectoralis minor
b- serratus anterior
c- teres minor
d- latissmus dorsi

31/ when u pushing automobile with shoulder flexion, elbow extended the
main muscle of action is:- TIBIALIS ANTERIOR
a-serratus anterior MUSCLE.
b-biceps
c-pectorals major
d-triceps

32/ To strengthen this muscle, you should:-


a. Maintain dorsiflexion and walking some steps on heel
b. Walking on toes
c. Sit at edge of plinth, rest foot on ground and fully dorsiflex the foot

33/ Which three muscles attach to the first cuneiform bone?


a- The anterior tibialis, the posterior tibialis, and the fibularis (peroneus) longus
b- The extensor digitorum, the flexor hallucis, and the fibularis (peroneus) longus
c- The anterior tibialis, the peroneus brevis, and the fibularis (peroneus) longus
d- The flexor carpi ulnaris, the flexor pollicis brevis, and the flexor pollicis longu

34/ In which location is the median nerve frequently or mostly entrapped:-


a. Forearm
b. Arm
c. Elbow
d. Wrist

35/ pt has median Nerve compression assist him by:-


a) Weak of medial 2 fingers & decrease grasping objects
b) Numbness of 3 ½ fingers & weak opponents ms

36/ ulnar nerve give supply to:-


a- whole hand
b- outer3 1/2 fingers
c- inner 1 +1/2 fingers

37/ What muscle make lateral rotation of Humeral :


a-Pectoral major
b-Tears minor
c-Tears major
d-Rotator cuff

38/ patient came to you with pain in the right buttocks and local tenderness in the tibial
tuberocity with pain in resistive knee flexion what is your diagnosis?
A)hamstring strain
B)hip arthritis
C)sports hernia
D)joint inflammation

39/ when evaluating forearm muscles,the therapist asks the patient to flex and abduct the
wrist.the therapist actually evaluating which of the following muscles?
a- flexor Carpi ulnaris
b- flexor Carpi radialis
c-flexor digitorum profundus
d- biceps brachii

40/ physiotherapist asks pt to flex the wrist and adduct it :-


a- flexor Carpi ulnaris
b- flexor Carpi radialis
c-flexor digitorum profundus
d- biceps brachii

41/ Shortness of whitch muscle cause limitation in shoulder abduction and latral rotation:-
a-pectoralis major
b-serratus ant
c-sub scapularis MUSCLES ADDOCTORS OF HIP JOINT
d-teres major

42/ All of the following are adductors of hip joint except:-


A. Gracilis
B. Pectineus
C. Sartorius

43/ winging of scapula due to weakness of:-


a-pectoralis major
b-serratus ant
c-sub scapularis
d-teres major

44/ injury to long thoracic nerve leading to weakness of:-


a-pectoralis major
b-serratus ant
c-sub scapularis
d-teres major

45/ with affection of phrenic nerve we stretch:-


a-sternocledomastoid
b-trapezius
c-diaphragm
d-masseter
46/ which muscle used in swimming:-
a- pectoralis minor
b- serratus anterior
c- teres minor
d- latissmus dorsi

47/ What is the action of this muscleshape ?


A- hip adduction
B- hip extenstion
C-back extension

48/All of the following muscles affected in tennis elbow except:-


a- Ext carpi radialis brevis 48/D:- muscles affected in tennis elbow
b – Ext carpi ulnaris
c – Ext digitorum
d- flex carpi ulnaris

49/what’s the name of this muscle:-


a- Biceps brachii
b- Brachioradialis

50/Patient suffering from pain insertion of this muscle the diagnosis:


a-rotator cuff tear
b-infra spinatus tendindinits
c-surpaspinatous tendinitis

51/ which of the following doesn't contribute to stabilizers of:-


a-sternoclavicular J
b-interarticular disc
c-ant. and pos. sc ligaments
d-acromiclavicular ligament

52/which part not included in posterior neck triangle:-


a-clavicle
b-trapezuis
c-strenoclidomastoid
d-sternum

53/ which is not related to the shoulder triangle:-


a)Clavicle
b)Sternum
c) Humorous
54/Protraction and retraction of shoulder occurs at:-
a-glenohumeral joint
b-scapulothoracic interfance
c-acromioclavicular joint

55/The physical therapist is reading the physician’s interpretation of an x-ray that was taken
of the left humerus of a 7-year-old patient. The physician notes in the report the presence of
an incomplete fracture on the convex side of the humerus. Which type of fracture is the
physician describing?
55/ C:-This scenario describes a greenstick fracture, which is common in
a-Comminuted
young people. In a comminuted fracture, the bone is broken into pieces. An example of an
b-Avulsion
avulsion fracture is when the tibial tuberosity is pulled off the tibia. A bone that has a
c-Greenstick
segmental fracture is fractured in two places.
d-Segmental

56/ Bennett’s fracture is:-


a. Little finger
b. Thumb
c. Middle finger
d. Distal end of radius

57/ A patient presents to an outpatient clinic with an order to evaluate and treat the right
forearm and wrist secondary to nerve compression. The patient has the following signs and
symptoms: pain with manual muscle testing of pronation, decreased strength of the flexor
pollicis longus and pronator quadratus, and pain with palpation of the pronator teres. What
nerve is most likely compromised? What is the most likely area of compression?
a-Median nerve – carpal tunnel
b-Ulnar nerve – Guyon’s canal
c-Ulnar nerve – pronator quadratus
d-Median nerve – pronator teres
57/D:- These signs and symptoms are common with median nerve compression as it travels through the two heads of
the pronator teres. Carpal tunnel syndrome usually presents with a positive Tinel's sign, a positive Phalen's test, and
decreased strength and sensation over the median nerve distribution. Ulnar nerve compression at Guyon's canal
typically presents with numbness pain, and tingling along the ulnar nerve distribution

58/ Thumb limited mov.which of the following ms.not affected:-


a Flex.Pollicis brevis
b Abd. P. brevis
c. oppons. Pollicis

59/ Which of the following is not part of the triangular fibrocartilage complex of the wrist?
a-Dorsal radioulnar ligament
b-Ulnar collateral ligament
c-Radial collateral ligament
d-Ulnar articular cartilage

60/ Which tendon is most commonly involved with lateral epicondylitis?


a-Extensor carpi radialis longus
b-Extensor carpi radialis brevis
c-Brachioradialis
d-Extensor digitorum

61/ There are post contusion of the patient thumb, the least movement allow:-
a- radial and ulner deviation
b- thumb adduction and flexion
c- extension and flexion of the wrist
d- supination and pronation

62/ You are performing sensory tests on a patient diagnosed with C6 nerve root
impingement. Testing should concentrate on the:
a-3rd, 4th and 5th fingers
b-ulnar border of the hand
c-thumb and index fingers
d-medial forearm

63/ The number of joint and bones in the foot:-


a-23 joint and 25 bones
b-33 joint and 26 bones
c-23 joints and 17 bones

64/ elbow joint:-


a- uniaxial
b-biaxial
c- three axial
d-multiaxial

65/ all of the follwoing are synovial joints except:-


a- Elbow joint
b- Knee joint
c- Hip joint
d- Symphysis pubis

66/ which is synovial joint:-


a- TMJ joint
b- Symphisis pubis

67/ during test of splaneous capitis which action not needed :


a) Extension
b) Flexion
c) Rotation

68/Splenius capitis response of :


a- neck flexion
b- neck extension
c- neck rotation
d- neck side bend

69/ which of following not needed in testing the trapezius ms:-


a-head flexion
b-head extension
c-head rotation
d-head lateral flexion

70/ which of following not needed in testing the sternocledomastoid ms:-


a-head flexion
b-head extension
c-head rotation
d-head lateral flexion

71/ A physical therapist is beginning an evaluation of a 5-year-old boy. The mother indicates
that she pulled the child from a seated position by grasping the wrists. The child then
experienced immediate pain at the right elbow. The physician’s orders
are for right elbow range of motion and strengthening. Which of the following
is the most likely diagnosis?
a-Radial head fracture
b-Nursemaid’s elbow
c-Erb’s palsy
d-Ulnar coronoid process fracture

72/ patient has hand injury without open wound and making tight band only after few 3
week referral to you and have contracture of hand and wrist causes of these are :
a- Dupuytren’s contracture Dupuytren’s contracture Volkaman contracture
b-Volkaman contracture (Tendon hand affected) (Tendons hand&wrist)

Dupuytren’s contracture is a progressive thickening of the palmar


aponeurosis
73/ Rheumatoid arthritis:-
a- cold limb
b- radial deviation
c- pain ,wet ,swelling

74/ The first thing affected in Rheumatoid arthritis :


a- capsule
b- articular cartilage
75/ Involvement of PIP joint, DIP joint and the carpometacarpal joint of base of thumb with
sparing the wrist is seen in:
a- Rheumatoid arthritis
c- Osteoarthritis
d- Psoriatic arthritis
e- Pseudo gout

76/ O.A in old age due to:


a- traumatic
b- tesr and wear process

77/ Sever old osteoarthritis patient. You recommended for:-


a- hot backs + exercise + walking
b- walkers
c-cold backs + exercise + walking
d-cane

78/ Pt complain of pain in the hip region from 3years increasing. Whith time &WB its:-
A- O.A of hip joint
B- Tight hip flexor
C- Trocantric bursitis

79/ Pt cannot raise hand in abduction or mentain abduction position due to:
a) Adhesive capsulitis
b) Supra spinatous tendenitis
c) Rotator cuff tear

80/ patient with shoulder pain during evaluation there is no passive or active ROM in abd.
and rot. That may caused by :
a- frozen shoulder
b- supraspinatus tendonitis
c- rotator cuff tear
d- acromioclavicular dysfunction

81/In taping an athlete’s ankle prophylactically before a football game, in what position
should the ankle be slightly positioned before taping to provide the most protection against
an ankle sprain?
a-Inversion, dorsiflexion, abduction
b-Eversion, plantarflexion, adduction
c-Eversion, dorsiflexion, abduction

82/ Acute adhesive capsulitis:


a- acute inflammation capsule with fibrous
b- chronic inflammation capsule with fibrous
c- capsule inflammation
d- fibrous capsule

83/ During examination of shoulder, you found increased motion (laxity) in anterior,
posterior and inferior directions, the condition is:-
a- Global shoulder instability
b- Rotator cuff tear

84/ A patient with a diagnosis of a rotator cuff tear has just begun active range of motion.
The therapist is strengthening the rotator cuff muscles to increase joint stability and oppose
the superior shear of the deltoid.Which of the rotator cuff muscles participate least in
opposing the superior shear force of the deltoid?
a-Infraspinatus
b-Subscapularis
c-Teres minor
d- Supraspinatus

85/ A therapist is attempting to gain external rotation range of motion in a patient’s right
shoulder. The therapist decides to use contract-relax-contract antagonist. In what order
should the following rotator cuff muscles contract to perform this movement successfully?
a-Infraspinatus – teres minor
b-Subscapularis – supraspinatus
c-Teres minor – infraspinatus
d-Supraspinatus – subscapularis

86/ in condromalacia consider in the ttt?


a- patellar tapping and strenthening of the quadriceps
b- strenthening vastus medialis only
c- strenthening vastus lateralis only
d- strenthening of hamstring

87/ myositis ossificans most affected


a- wrist Joint ‫معلومة ع الماشي‬
b- Elbow Joint -young patient with post fracture elbow deformity
c- Shoulder Joint referred for pt what's contraindicated
d- Knee Joint to avoid myositis ossificans vigorous passive stretch

88/ Spyndolythesis:-
a-vertbera above vertebra dislocat posterior
b-vertbera below vertebra dislocat posterior
c -vertbera above vertebra dislocat anterior
d-vertbera belowe vertebra dislocat anterior

89/ osteomyelities is:-


a-infective inflammation of the bone due infective bacteria entered into bone
b-decreased bone density and increased porosity
d-due to repetitive trauma
c-vit D deficiency

90/A patient presents to physical therapy with complaints of pain in the right hip due to
osteoarthritis. Which of the following is not true about this type of arthritis?
a-Causes pain usually symmetrically because it is a systemic condition.
b-Not usually more painful in the morning.
c-This type of arthritis commonly involves the distal interphalangeal joint.
d-Mainly involves weight-bearing joints.
90/Choice A describes rheumatoid arthritis, a systemic
condition. All of the other choices are signs and symptoms
of (OA). Sometimes OA can involve symmetric joints,
but it is not systemic.
91/ A 65 years old pt wuth well treated collas fracture ,few monthes later came with sever
pain in hand with coldness color changes . x ray show big callus around radial head???not
remember This may be due to
A- osteoarthritis of wrist
B -prearticular ossification ‫ثقف نفسك‬
Sudeck's atrophy/reflex sympathetic
C- Sudeck's atrophy dystrophy (RSD)/algodystrophy/causalgia
The term complex, regional pain syndrome is now being
92/ Sudeck's atrophy which is not true:- used to describe these pathological states. This is a
complication where the patient complains of severe pain
a- sever burning pain on movement, or at rest, out of proportion to the initial
b- sympathetic hyperactivity injury. The limb is swollen. The skin appears shiny and
c- trophic changes occur discoloured and feels cold; in extreme cases this may lead
d- more perspiration to the limb becoming exquisitely tender and discoloured.
Osteoporosis and permanent contractures.

93/ which is not true about sudeck's atrophy:-


a- osteoporosis of bones of hand
b- fully curable by taking sympathetic nerve blocker

94/ what is the cause of hip dysplasia?


A)mother malnutrition
B) idiopathic

95/ Following the reduction of developmental dysplasia of the hip in an infant, the physical
therapist should instruct the parents to maintain that hip in a stable position of:-
a-flexion and adduction
b-flexion and abduction
c-extension and adduction
d-extension and abduction

96/ All is true about med. meniscus except:-


a- More injured than lat.men
b- Absorb of about 90% of load on knee joint
c- Injury from hyper ext injury

97/ History of foot ball players has injured by twisted knee and take out of game and after
little time it swelling and warm . after few days it locks during climbing stairs and painful and
cannot put full weight in walking :
97/C:- MCl rupture due to
a- patella fracture
it`s injured by twisted
b- tibia fracture
knee , and there is
c- MCL rupture
ascending pain with
d- Rupture of something of fibers
swelling , warm and locks
in some degree of flexion.

98/years old foot player come to the physiotherapist with pain at the left knee joint while
the physiotherapist taking history patient said that the injury occurred when he rotates to
the right on weight bearing to the left knee .it was flexed 60 degree. that means that the
injury is at :-
a- medial collateral ligament semilunar cartilage = meniscus
b- semilunar cartilage at the knee
c- femoral condyles
d- upper shaft of the tibia
99/ A patient is an outpatient facility because of an injury sustained to the right knee joint.
Only the structures within the synovial cavity were compromised during the injury. Knowing
this information only, the therapist is not concerned with injury to which of the following
structure?
a-Patellofemoral joint 99/B:-The anterior cruciate ligament is located within the articular cavity
b-Anterior cruciate ligament but outside the synovial lining. The anterior and posterior cruciate
c-Medial meniscus ligaments have their own synovial lining.
d-Femoral condyles

100/ What portion of the adult knee meniscus is vascularized?


a- Outer edges
b- Inner edges
c- The entire meniscus is vascular
d- The entire meniscus is avascular

101/ delayed ulnar nerve due to fracture of:-


a- olocranion fossa
b- medial epicondyle 103/A:- Whiplash is a nonmedical term used to
c- lateral epicondyle describe neck
*pain following an injury to the soft tissues of
your neck (specifically ligaments, tendons, and
102/Hyaline cartilage innervated by:- muscles). It is caused by an abnormal motion or
force applied to your neck that causes
A -blood vessels movement beyond the neck's normal range of
b- synovial fluid motion.
c- epiphyseal growth symptoms of whiplash injury are:
•Neck pain
•Neck swelling
103/Acceleration injury (whiplish injury) may cause:- •Tenderness along the back of your neck
•Muscle spasms (in the side or back of your
a- pain in posterior aspect of neck neck)
b- pain at post aspect of back •Difficulty moving your neck around
c- dislocation of vertebral colum •Headache
•Pain shooting from your neck into either
d- tearing of post para spinal ligament shoulder or arm

104/The most common site of fracture in old patient due to failing is:-
a- head of femur
b- trochanteric
c- shift of femur
d- neck of femur
106/Boutonniere deformity is a deformed
position of the fingers or toes, in which the
joint nearest the knuckle (PIP) is permanently
105/ All of the following is clinical features in fracture neck of femur bent toward the palm while the farthest joint
(DIP) is bent back away (PIP flexion with DIP
except:- hyperextension). It is commonly caused by
a-pain in hip region injury[1] or by an inflammatory condition like
rheumatoid arthritis, or genetic conditions like
b- abduction of leg Ehlers Danlos Syndrome.
c-shortness of the limb
d- lateral rotation of the leg

106/ The therapist receives a referral to evaluate a patient with a


boutonniere deformity. With this injury, the involved finger
usually presents in the position of:-
a- Flexion of the proximal interphalangeal (PIP) joint and flexion
of the distal interphalangeal (DIP) joint.
b- Extension of the PIP joint and flexion of the DIP joint.
c- Flexion of the PIP joint and extension of the DIP joint.
d- Extension of the PIP joint and extension of the DIP

107/A boutonniere deformity of the finger would not consist of:-


a-hyperextension of the proximal interphalangeal joint
b-overstretch of the extensor digitorum communis tendon
c-volar slippage of the lateral bands
d-hyperextension of the distal interphalangeal joint

108/a patient referred to physical therapy with


a diagnosis of Dupuytren’s contracture?
Which of the following is used to treat
A-Knee continuous passive motion (CPM)
b-Work simulator set for squatting activities.
c-Hand splint.
d-Atwo-pound dumbbell

109/avascular necrosis can be a possible Sequelae of FRACTURE of all of the following bones,
except:
a-Femur neck
b-Scaphoid
c-Talus.
d-Calcaneum

110/player football take trauma in anterior aspect of tibia with positive posterior drawer
sign:-
a- anterior cruciate ligament
b-posterior cruciate ligament
c-medial collateral ligament
d- lateral collateral ligament

111/Avascular necrosis would least likely


be a major complicating factor in the reha-
-bilitation of a patient with:
a-Legg-Calvé Perthes’ disease
b-a tibial fracture
c-degenerative joint disease of the cervical spine
d-a carpal scaphoid fracture
LCPD) Perthes disease-Calve-Legg
resulting avascular necrosis of the proximal femoral headis from compromise of the tenuous blood supply to this area. LCPD
usually occurs in children aged 4-10 years. The disease has an insidious onset and may occur after an injury to the hip. In the vast
majority of instances, the disorder is unilateral. Both hips are involved in less than 10% of cases, and the joints are involved
successively, not simultaneously
BLOOD SUPPLY TO MUSCLE DURING EX
The cause of cubital valgus fracture of(lateral condyle Fracture)
The cause of cubital varus fracture of (supracondylar Fracture)
The position of mechanical traction for lumber disc herniation (prone with no pillow)
The sheath of mucle fiber its name (sarcolema)
paraesthesia means(Destruction in sensory system cause abnormal sensation)
Femoral nerve stretch test –(‫ )كتير ويكرر جدا مهم‬the
patient lies prone, the knee is passively flexed to the thigh and the hip is passively extended; the test is positive if the patient
experiences anterior thigh pain. This test is usually strongly
positive in patients with protrusions at L2-L3 and L3-L4, slightly
positive or negative in L4-L5 disk protrusions and negative in
cases with a lumbosacral protrusion
112/To prevent maximal compressive forces being placed on the patella, a therapist should
minimize placing the patient:
A. prone and flexing the knee to 30 degrees
B. in a sitting position with the knee flexed to 90 degrees
C. supine and flexing both the hip and knee to 110 degrees
D. prone and flexing the knee to 110 with the hip extended

113/Knee medial rotation due to:-


A.weak vastus medialis
B.weak semitendinosis
C. Weak quadriceps
d-weak bicepis femoris

114/Full adductionDuring evaluation of a patient, the therapist observes significant posterior


trunk lean at initial contact (heel strike). Which of the following is the most likely muscle that
the therapist needs to focus on during the exercise session in order to minimize this gait
deviation?
a-Gluteus medius
b-Gluteus maximus
c-Quadriceps
d-Hamstrings, full internal rotation and full extension

115/Decrease osteoblast cause:-


a-osteoporosis
b-ricket
c-osteomalacia

116/When the ankle is forcibly inverted and plantar flexed,


the ligament that is most frequently disrupted is the:
a-deltoid
b-anterior talofibular
c-posterior talofibular
d-calcaneofibular

117/increase glenoid cavity for proximal humerus in shoulder j.


A.capsule
B.labrum
C.lig. And tendon

118/Inversion injury at the ankle can cause all of the following except
a- FRACTURE tip of lateral melleolus
b- FRACTURE base of the 5th metatarsal
c- Sprain of extensor digitorum brevis
d- FRACTURE of sustentaculam talis

119/A45years male presented with an expansile lesion in the centre of


femoral metaphysis. The lesion shows endosteal scalloping & punctuate
calcifications. Most likely diagnosis is:-
a- Osteosarcoma
b- Chondrosarcoma
c- Simple bone cyst
d- Fibrous dysplasia

120/most common site of osteogenic sarcoma is:


a-Femur, upper end
b-Femur, lower end 120/ A and B correct (not sure)
c-Tibia, upper end
d-Tibia, lower end

121/Deformity associated with coll’s fracture ( fracture lower end of radius):-


a- Dinner fork deformity
b - Pes planus
c- Talipus equinovarus

122/ Coll`s fracture may possibly cause injury to which of


the following:-
a-flexorpollicis
b-abductor pollicis
c-adduuctorpollicis
d-extensor pollicis longus

123/What the other name of fracture radius:-


a-no name
b-pontine fracture
c-bennt fracture
d-colles fracture

124/which is in correct about plump line of body in standing:-


A-.pass post. To ear lobe
B-.pass through shoulder
C-.pass Posterior to greater trochanter
D- pass anterior to ankle

125/One of the possible complications following a fracture is Volkmann's ischemic


contracture. This condition:
a- Is caused by an interference with the venous return
b- Is caused by an interference of the nerve supply )LOG( ‫ببساطة كي ال تنسى‬
c- May occur if the fracture is sustained in the upper extremity ‫) الكتف والكعبين‬2(‫بيعدي امام‬
d- None of the above ‫) الفخذ واالذنين‬2(‫وبيعدي ورا‬
‫) الركبة والفقرة التانية‬2( ‫وبيعدي جوا‬
126/Which is correct about volkmans ischemic contracture: ‫في اللمبرر‬
a-affect flexor of forearm
b-affect palmar fascia
C-lead to ulnar neuritis

127/volkmans ischemic contracture occurs due to:


a-fracture of medial epcondyle of humerus
b-fracture of lat epicondyle of humerus
c-fracture lower third of humerus
d- surgical neck
128/which is not correct about Gallilze # :-
A-fracture of raduis and sublaxtion of ulna
b- fracture of ulna and sublaxtion of raduis
c-its occur by falling

fracture at surgicalneckofhumerus -injury of Axillary nerve & muscle affected Deltoid and occur loss of abduction of arm
fracture at midshaft (spiral groove) of humerus-injury of Radial nerve & occur wrist drop & decreased extension of elbow
fracture at medial epicondyle of humerus -injury of Ulnar nerve & occur Ulnar claw hand
fracture of distal radius ( resulting from fall on outstretched hand (extended wrists) produces "dinner fork" appearance where distal fragment is
posterior )
Colles' fracture fracture of distal radius ( resulting from fall on back of hand (flexed wrists) less common ) -Smith's fracture most commonly
FRACTURED carpal bone ( avascular necrosis of proximal part when deprived of its arterial supply )
Scaphoid most commonly DISLOCATED carpal bone ( occurs in young adults )
Lunate Galezzi fracture -fracture radius & dislocation ulna
Montagia fracture -fracture ulna & dislocation radius

129/ Monteggia fracture means which of the following:-


a- fracture of the radius and sublaxation of the lower end of ulna Galeazzi Fracture
b- fracture of the ulna with radial head sublaxation
c- green stalk fracture with minor angulationfracture of the radioulnar joint

130/All of the following are associated with supracondylar of humerus, except:


a- It is uncommon after 15 years of age
b- Extension type FRACTURE is more common than the flexion type
c- Cubitus varus deformity commonly results following malunion not sure
d- Ulnar nerve is most commonly involved

131/# valgus deformity results from:-


a-lateral condyalar-#
b- medidal condyalar-# lateral condyles # lead to valgus deformity
supracodylar # lead to varus deformity
c- supra condylar #

132/# Supracondyler ridge of humerus injury to:-


a- Ulnar n nerve
b-brachial nerve
c- median nerve Supracondyler #
133/Person fall onlat aspect of shoulder last thing occur is:-
a- dislocation sc joint
b- dislocate acromio clavicular g=joint
c- # medial cdlavicle
d- #middle third of clavicle

134/skeletal traction used as ttt of:-


a) # shaft of femur
b) # tibia

135/when the fracture accurse in neck of humerus what nerve affected:-


A-Redial N
B-Median N
C-Ulnar N
D-Subcotenuis nerve

136/this angle in the picture means:-


a-genovalgum
b-coxavalgum
c-coxavara
d-genurecrvatum

137/The angle between Neck of femur and Shaft of femur :


A- 90 Degree
B- 120 Degree
c-150 Degree
d-170 Degree

138/The angle between longitudinal axis passing throw fermal


neck and longitudinal axis of fumer is decreased in the case of :
A- genu valgum
B -genu varum
C- genu recurvatum
D- coxa vara

139/ the normal angle between the fumer and the neck of fumer is 126. when the angle
increase the deformity is:
a-coxa valga
b-genu Valgus
C- genu recurvatum

140/Coxa vara present as abnormality of:-


a-proximal femur
b-distal femur
c-proximal tibia
d-distal tibia

141/Most common compression of radial n in:-


1-shaft of humerus
2-neck of humerus
3-distal end

142/Congenital dislocation occur in:-


a- Hip joint
b- knee joint
c- shoulder joint
d- elbow joint

143/which statement not correct about fracture neck of femur:-


a-its fatal
b-need arthoplasty
c-lead to avascular necrosis
d-occur mostly in children

144/About Tennis elbow all the following is true except:-


A also called lateral epichondylitis
B precipated by ulnar neuritis
C usually occur in dominant side

145/paralysis of all intrinsic muscles of hand except abd poll brevis:-


A -median n
B -ulnar n
C -Radial n

146/most compression site of superfacial peroneal nerve is :


a) Latral condyle of femure
b) Latral head of fibula

147/Pt with lordosis from standing and +ve Thomas test dueto
a- strong lumbar ext
b- fixed flex deformity
c- hamstring strain

148/What is the most likely cause of anterior pelvic tilt during initial contact (heel strike)?
a-Weak abdominals
b-Tight hamstrings 148/a. Abdominal muscles attach to the lower border of the ribs and
c-Weak abductors the superior surface of the pelvis. Strong abdominals prevent excessive
d-Back pain anterior rotation of the pelvis during gait.

149/If the line of gravity is posterior to the hip joint in standing, on what does the body first
rely to keep the trunk from moving into excessive lumbar extension?
a-Iliopsoas muscle activity
149/C-In static standing, the line of gravity is
b-Abdominal muscle activity
posterior to the hip joint. The body relies on the
c-Anterior pelvic ligaments and the hip joint capsule. anterior pelvic ligaments and the hip joint capsule.
d-Posterior pelvic ligaments and the hip joint capsule. The iliopsoas may be recruited at times, but
anterior ligaments are used first to keep the trunk
150/ sciatic nerve injury due to:- from extending in static stance.
a) Post dislocate hip
b) Ant dislocate hip
151/In contracture of lower lumbar back muscles, the pelvis is affected by:-
a. Anterior pelvic tilting with upper thoracic kyphosis
b. Anterior pelvic tilting with lordosis
c. Posterior pelvic tilting with lordosis
d. No effect on the pelvis

152/Patient with abd scapula,round shoulder .,kyphosis ,ant rotation of pelvic


A. Lordosis
B. Sway back
C. Flat back
D. Kypholordosis

Flat back__ post pelvic tilt+ hyper ext knee


Sway back__ post pelvic tilt+ hyper ext knee ‫مع زيادة‬hyper ext hip
(Kyphosis(‫ بيحصل في الـ‬thoracic
Lordosis__ Ant pelvic tilt +slight hyper ext knee.
Hyper Lordosis__ Ant pelvic tilt +slight hyper ext knee.

153/Pt has post pelvic tiled , flat lower thoracic & increase upper back kyphosis:-
A) Flat back
B) Sway back

154/ The therapist performed Trendelenburg test for patient with right
weakness of gluteus medius. The patient stands on right lower limb.
The pelvis drop would be observed as:-
a. Pelvis does not drop as the patient leans by the trunk to the right
b. Pelvis will drop at the left side
c. Pelvis will drop at the right side
d. Pelvis does not drop as the patient leans by the trunk to the left

155/The therapist receives a referral to evaluate a patient with a boutonniere deformity.


With this injury, the involved finger usually presents in the position of:-
a-Flexion of the proximal interphalangeal (PIP) joint and flexion of the distal interphalangeal
(DIP) joint.
b-Extension of the PIP joint and flexion of the DIP joint.
c-Flexion of the PIP joint and extension of the DIP joint.
d-Extension of the PIP joint and extension of the DIP joint.

156/f there is weakness of right gluteus medius dropping pelvis of occurs at:-
a-right pelvic drop at stance phase
b-left pelvic drop at stance phase
c-right pelvic drop at swing phase
d-left pelvic drop at swing phase

157/A therapist is evaluating a patient with poor motor coordination. The therapist observes
that when the patient is standing erect and still, she does not respond appropriately when
correcting a backward sway of the body. With the body in a fully erect position, a slight
backward sway should be corrected by the body firing specific muscles in a specific order.
Which list is the correct firing order?
A-Bilateral abdominals, bilateral quadriceps, bilateral tibialis anterior.
B-Bilateral abdominals, bilateral tibialis anterior, bilateral quadriceps
C-Bilateral tibialis anterior, bilateral abdominals, bilateral quadriceps
D-Bilateral tibialis anterior, bilateral quadriceps, bilateral abdominals.

158/Which of the following is observed by the therapist if a patient is correctly


performing an anterior pelvic tilt in standing position?
a-Hip extension and lumbar flexion.
b-Hip flexion and lumbar extension.
c-Hip flexion and lumbar flexion.
d-Hip extension and lumbar extension

159/ Which of the following is a possible cause of anterior pelvic tilting:-


a. Weak lateral trunk muscles
b. Tightened tensor fascia lata
c. Tight hamstring
d. Severe weakness of abdominal muscle

160/If the line of gravity is posterior to the hip joint in standing, on what does the body first
rely tokeep the trunk from moving into excessive lumbar extension?
a- Iliopsoas muscle activity
b- Abdominal muscle activity 160/C:- In static standing, the line of gravity is posterior
to the hip joint. The body relies on the anterior pelvic
c- Anterior pelvic ligaments and the hip joint capsule.
ligaments and the hip joint capsule.The iliopsoas may be
d- Posterior pelvic ligaments and the hip joint capsule. recruited at times, but anterior ligaments are used first
to keep the trunk from extending in static stance.
161/ Right convex Scoliosis we find:-
A- Lateral trunk flexion & concave side to RT
B- Lateral trunk flexion &convex side to RT
C- Lateral trunk flexion &convex side to LT
D- Lateral trunk flexion & shorten RT musculature

162/A 14-year-old girl with right thoracic scoliosis is referred to physical therapy. The
therapist should expect which of the following findings?
a-Left shoulder high, left scapula prominent, and right hip high
b-Left shoulder low, right scapula prominent, and left hip high
c-Right shoulder high, right scapula prominent, and right hip high
d-Right shoulder low, right scapula prominent, and left hip high
163/The most common deformity in geriatric is:-
a-Kyphosis
b-Scoliosis
c-Kyphoscoliosis
d-Lordosis

164/Severe kyphoscoliosis will lead to:-


a- Left rather than right ventricular failure
b- Right rather than left ventricular failure
c- Frequent respiratory infections
d- No cardiac abnormalities

165/You are evaluating an athlete who is complaining of pain in the left shoulder region.
Your assessment of the shoulder elicits pain in the last 30 degrees of shoulder abduction
range of motion. this finding is most congruent with:-
a-calcific supraspinatus tendinitis.
b-subacromial bursitis.
c-acromioclavicular sprain.
d-thoracic outlet syndrome.

166/Which of the following is the most vulnerable position for dislocation of the hip?
a-30o hip extension, 30o hip adduction, and minimal internal rotation
b-30o hip flexion, 30o hip adduction, and minimal external rotation
c-30o hip flexion, 30o hip abduction, and minimal external rotation.
d-30o hip extension, 30o hip abduction, and minimal external rotation.

166/C. This is the loose-packed position of the hip

167/A patient with adhesive capsulitis of the glenohumeral joint should demonstrate
the greatest limitation of motion when performing shoulder:-
a-flexion.
b-abduction.
c-medial rotation.
d-lateral rotation.

168/A therapist performs a test for a patient and was positive with the thigh of the patient
raised some inches above the examination table. What is the test name and which muscle is
shortened?
a. Ober test – Iliotibial band
b. Straight leg raising test – hamstring
c. Thomas test – Iliopsoas
d. Thomson – gastrocenmies
169/PT Lachaman to asses:-
A- A.C.L
B- M.C.L
C- P.C.L

170/Pt has tear with rotation movement :-


Which test not need :
a- drawar
b- lackmen
c-trendburg

171/A patient is placed in supine position with the knee in 90o of flexion. The foot is
stabilized by the therapist’s body on the examination table. The therapist then wraps his
fingers around the proximal tibia so that the thumbs are resting along the anteromedial and
the anterolateral margins. The therapist then applies a force to pull the tibia forward. What
special test is being performed?
a-Pivot shift
b-Lachman’s test
c-Anterior drawer
d-Posterior drawer

172/A therapist is examining a 3-year-old child who is positioned as follows: supine, hips
flexed to 90 o ,hips fully adducted, and knees flexed. The therapist passively abducts and
raises the thigh, applying an anterior shear force to the hip joint. A click at 30 o of
abduction is noted by the therapist. What orthopedic test is the therapist performing, and
what is its significance?
a-Ortolani’s test – hip dislocation
b-Apley’s compression/distraction test – cartilage damage
c-McMurray test – cartilage damage
d-Piston test – hip dislocation.

172/a. Ortolani's test is used to detect a congenitally


dislocated hip in an infant. Choices B and C are common
meniscus damage tests for the knee. Choice D is performed by
placing the infant in supine position with the hip at 90 degrees
of flexion and slight abduction and the knee flexed to 90
degrees. The examiner then moves the infant's hip anterior
and posterior in an effort to detect abnormal joint mobility.

173/ patient has shoulder dislocation and reduction for this dilocation and referral to you
for early mobilization to prevent :
a- Stiffness
b- Recurent dislocation
C- Osteoarthritis
174/ Fracture due to repeated minor injury:
a- pathological
b- fatigue
c- reapeated stress
d- compression

175/ Pt has cruciate ligament injury:-


a) Strength Quad
b) hamstring
c) Both

176/posterior shear test for:-


a- sacroilliac joint
b- hip joint
c- lumbosacral join

177/The main ms responsible for climbing stairs and coming from reclined position,
bringing knee to chest:-
a-rectusfemoris
b-iliopsoas
c-quadratus lumborum
d-pectineus

178/All the following is true about ligaments except :-


A- high collagen content
B- laxity lead to hyper mobility
C- connect on bone to another
D- usually heals fast and need not support

179/ Thompson test is for:-


a. Shortening of iliotibial band
b. Integrity of Achilles tendon
c. Shortening of iliopsoas
d. Integrity of quadriceps tendon

180/Talipes equinovarus:-
a. Ankle dorsiflexed, everted, adducted
b. Ankle plantarflexed, inverted, abducted
c. Ankle dorsiflexed, everted, abducted
d. Ankle plantarflexed, inverted, adducted

181/A therapist is scheduled to treat a patient with cerebral palsy who has been classified
as a spastic quadriplegic. What type of orthopedic deformity should the therapist expect
to see in a patient’s feet?
A-Talipes equinovalgus
B-Talipes equinovarus
C-Clubfeet
D-B & C are correct
182/ Valgus position of foot:-
a-planterflexion
b-dorsiflexion
c-eversion
d-inversion

183/Maximum stability of hip joint depends on:-


a-bone,ligament,muscles
b-muscles,ligament,bone
c-bone only

184/Ulnar nerve pass:-


a- anterior to medial epicondyle
b- anterior to lateral epicondyle
c- posterior to medial epicondyle
d- posterior to lateral epicondyle

185/The shoulder work with 3 joint except:-


b- acromioclavicular
c- scapulothoracic
c- sternoclavicular

186/which of the following you cannot palpate while examining shoulder:-


A-2nd rib
B-Sternoclavicular joint
C-1st rib

187/ patient came to clinic with morning pain and stiffness this disease mean:-
a-systemic degenerative
b-ms spasm
c-joint infection

188/A 13-year-old girl has fractured the left patella during a volleyball game. The physician
determines that the superior pole is the location of the fracture. Which of the following
should be avoided in early rehabilitation?
a-Full knee extension
b-45 o of knee flexion
c-90 o of knee flexion
d-15 o of knee flexion

189/A patient is referred to the therapist with a diagnosis of arthritis. What type of arthritis
would the therapist expect if the patient present with the following signs and symptoms? (1)
Bilateral wrists and knees are involved, (2) pain at rest and with motion, (3) prolonged
morning stiffness, and (4) Crepitus.:-
a-Osteoarthritis 189/b:-. Rheumatoid arthritis is a systemic
b-Rheumatoid arthritis condition commonly involving joints bilaterally
c-Degenerative joint disease Crepitus can be associated with osteoarthritis or
d-It is not possible to determine with the given information rheumatoid arthritis, but rheumatoid arthritis is
the most likely in this case
190/ The difference between RA and OA:-
a- OA's degenerative disease
b- b- RA's is systemic disease & and joint have deformity
c- RA's remission and relapse d- OA's curable disease

Because hydro static pressure on chest wall and on abdomen wall which acts on diaphragm and
there is indirect effect on the shift of blood to the thorax :resulting in decrease vital capacity

191/In degenerative joint disease which is not occure?


a-increased with weight bearing on the joint
b-gradual onset
c-stiffness at morning
d-increaser after prolonged period of unactivity

Characteristics of degenerative joint disease Signs and symptoms may include: • pain that
increases on weight bearing activities (standing and walking, walking downstairs particularly).
• insidious onset of symptoms
followed by progressive periods of relapses and remissions.
•pain and stiffness in the morning.
• stiffness following periods of inactivity.
•pain and stiffness that arise after unaccustomed periods of activity.
• bony deformity (e.g. characteristic varus deformity may follow from collapse of
the medial compartmental joint space).
• reduction of the joint space observed on X-ray, with bony outgrowths or osteophytes

ankylosing means : ankylosing spondylitis mean : known as Bechterew's disease (or syndrome) and Marie- Strümpell
disease, is a chronic inflammatory disease of the axial skeleton with variable involvement of peripheral joints and
nonarticular structures Also Ankylosing is a term meaning stiff or rigid and spondylitis means inflammation of the
spine .

Joints Types:
Hip Joint__ Ball-and-Socket
Knee joint__ Bicondylar joint
Ankle Joint__ hinge joint
Sternoclavicular Joint__ Saddle Joint
Acromioclavicular Joint__ Plane Joint
Shoulder/Glenohumeral Joint__ Ball and Socket Joint
Elbow Joint__ Hinge Joint
Wrist/Radiocarpal Joint__ Condyloid Joint
Inercarpal and Midcarpal joint__ Plane joints
Carpometacarpal Joint of the thumb__Saddle
Carpometacarpal joint of the fingers__Plane joints.

192/When you examine shoulder joint by asking patient


To abduct shoulder to 90 degrees then lower slowly, this is:
a- Codman’s Test ( for rotator cuff tear) ( also called drop arm test)
b- Infraspinatus test

193/Patient stay at hospital treated by corticosteroid then


go to physiotherapist, this patient will have problem it is:-
a- myopathy
b- osteoprosis
c- deprssion
d- decrease apeptit
194/patient has amputation and use artificial limb during walking he takes abduction gait,
this may due to:-
a- limb is high
b- stress on adductor longus muscle
c- medial aspect is short
d- tight of gluteus medius

195/when examine patient for carpal tunnel syndrome,which nerve do you examine:-
a- Radial n.
The muscles that might be affected by CTS are the "LOAF" muscles...lumbricals,
b- Ulnar n. opponens (opposes thumb to other fingers), abductor pollicis brevis (thumb abduction),
c- Median n and flexor pollicis brevis (flexes thumb). **5th finger abduction and adduction of the
thumb are mediated by ulnar nerve muscle.

196/ Disease in children and early childhood due to malfunction of calcium phosphorus
metabolism:-
a-rickets
b-osteomyelitis

197/Most compression site of ulnar nerve in:-


a-shoulder jt
b-spinal groove of humerus
c-elbow
d-wrist

198/In carpal tunnel syndrome the entrapped nerve is the?


a-radial
b-median
c-ulnar
d-axillary.

199/which muscle doesn't affected in carpal tunnel syndrome:-


a-flexor pollicis brevis
b-adductor pollicis longus
b-abductor pollicis bervis

200/which movement not accompany with hyper extension of cervical region:-


a-poking chin
b-decrease flexion of lower cervical region
c-increase activity of SCM AND elevator scapulae

201/what is the best position for artificial hip ankylosis:-


A-Flex 20 , abd 10, ext rotation 5
B-No flex , abd 10, ext rotation 5 201/Surgical Considerations: Arthrodesis, also known as artificial
C -Abd 5 , ext rotation 10 ankylosis
- position of hip fusion:
202/position of hip arthroplasty:- - neutral abduction, external rotation of 0-30 deg &, 20-25 deg of
flexion.
a-abd. 30 with flex. maintained at 70_90 - avoid abduction and internal rotation;
b-abd.20 with flex. maintained at 70_90 - this position is design to minimize excessive lumbar spine motion.
c-abd.10 with flex maintaINED at 70_90
d-abd.30 with flex. maintained at 20_90
203/Patient with depression of metatrsal pad and claw toe the clinical picture:
a-Hyperext of metatarsal bone and ext of I.P.JT
B-FLEXION of metatarsal bone and ext of I.P JT Clawing (hyperextension of the
metatarsophalangeal joints and flexion
C-ABDUCTION OF metatarsal bone AND FLEXION I.PJT of the other phalanges)
D-hyper ext of metatarsal bone and flexion of I.P JT • mallet toe (flexion of the distal
interphalangeal joints)
204/WHICH STATEMENT NOT CORRECT •hammer toe (hyperextension of the
ABOUT TALIPES EQUINO VARUS? metatarso-phalangeal and flexion of the
proximal interphalangeal join)
A-club foot
•hallux valgus (lateral deviation of the
b-maybe congenital first interphalangeal)
c-p,f and inversion • hallux rigidus (stiffness of the first
d- d.f and enversion interphalangeal)

205/pt is refferd to physio dept with diagnosis of flexion deformity of rt knee with
examination we will found:-
A flexion knee +planter flexion +shortening of qudricepes
B flexion knee +dorsi flexion +shortening of hamstring
C flexion knee +no ankle changes +shortening of quadriceps
D flexion knee +planter flexion +lenghing of soles

The symptoms are variable and will present themselves differently from patient to patient. The main symptoms begin with a
generalized burning pain; this is usually followed by changes in the condition of the skin, which may become shiny. In severe
cases, the affected body part may swell and, due to sympathetic nervous system dysfunction, the body part may perspire
more than usual. Because of the pain the patient may not want to move the injured part. This leads to muscle wastage and a
viscious cycle where stiffness and pain become worse. If the condition persists there may be adverse changes to the condition
of the underlying bone.

206/patient with winged scapula and excessive internal rotation needs to:-
a- stretch middle and lower trapezius
b- Stretch pectoralis major and strength middle trapezius
c- strength pectoralis major and Stretch upper trapezius

207/patient has scoliosis c on rt thoracolumbar we found:-


a-lat flexion of thoracolmbar on rt convex
b-lat flexion and lt convex
c-lat flex and concave on lt side

208/anterior taleofibular ligament to be assessed resist:-


a- planter flex + inversion
b- dorsi + inv
c- dorsi + eversion
d- plant + ev

209/What ligament is most involved in sustaining the longitudinal arch of the foot?
a-Plantar calcaneonavicular ligament
b-Long plantar ligament
c-Plantar calcaneocuboid ligament
d-Anterior talofibular ligament

210/Which of the following is not an example of a synarthrodial joint in the body?


a-Coronal suture
b-The fibrous joint between the shaft of the tibia and fibula
c-Symphysis pubis
d-Metacarpophalangeal

211/Ice massaging is contraindicated with:-


a- Inflammation
b- Reynaud's disease
c- Muscle spasm
d- Acute burn

212/A patient with Osgood-Schlatter disease will experience the most difficulty and pain
while:-
a-playing basketball or volleyball
b-sitting for prolonged periods of time
c-walking down stairs.
d-lifting heavy weights

213/A teenager comes to an outpatient facility with complaints of pain at the tibial
tubercle when playing basketball. The therapist notices that the tubercles are abnormally
pronounced on bilateral knees. What condition does the patient most likely have?
a- Jumper’s knees. 213/Osgood –Schlatter disease or syndrome (also
b- Anterior cruciate ligament sprain. known as Apophysitis of the tibial tubercle, or knobby
c- Osgood-Schlatter disease. knees) is an irritation of the patellar ligament at the
d-sever's disease tibial tuberosity.[1] It is characterized by painful
lumps just below the knee and is most often seen in
young adolescents. Risk factors may include
overzealous conditioning (running and jumping), but
adolescent bone growth is at the root of it.
Osgood-Schlatter disease is severe tendinitis of the
patellar tendon. It is characterized by pronounced
tibial tubercles. Toe increased size of the tubercles is
attributed to the patella tendon pulling away from its
insertion. Jumper's knees ( or normal patella
tendinitis) does not necessarily present with tubercle
enlargement. Sever's disease involves the Achilles
tendon pulling away from its insertion
on the calcaneus

214/A tennis player receives a surgical repair of the annular ligament. Where should the
therapist expect to note the most edema?
a-Radial ulnar joint
b-Olecranon bursa
c-Ulnohumeral joint
d-Lateral triangle

215/foot deformity (hallux valgus)


.‫اذا جاك السؤال بيجيبلك نفس الصورة ويقولك اذكر المرض‬
216/ A patient is referred to physical therapy with a ruptured Achilles tendon. The patient
reports that he was moving the lawn and was going down a steep incline when he felt a
sharp, sudden pain in the left heel region.Which of the following would be proper location
for the Achilles tendon insertion?
a- Talus
b- Calcaneus
c- Cuboid
d- Navicular

217/What is true about this muscle?


A-this is biceps femoris ms. &supplied by tibial nerve
B-adductor brevis muscle
C-sartorius

218/triple arthrodesis involves:


a-Calcaneocuboid, talonavicular and talocalcaneal.
b-Tibiotalar, calcaneocuboid and talonavicular.
c-Ankle joint, calcaneocuboid and talonavicular.
d-None of the above
*The talonavicular (blue arrow)
* subtalar (red arrow)
219/Patient has chondromalacia chontra indicated to use? * calcaneocuboid (green arrow)
a-mobilization joint.
b-taping , bandage
c-strengh hamstring

220/Which of the following articulate with the second cuneiform?


a-Navicular
b-Talus
c-First metatarsal
d-Cuboid

221/patient come with sever elbow pain we can use:-


a- Heat application
b- Cold application if acute trauma use cold & if chronic use heat & if acute inflamation so use cold
c- Faradic stimulation

222/Closed pack position of knee is:-


a-ext knee
b-flex knee
c-ext knee with lat tibial rotation

223/What is the closed-packed position of the shoulder?


a. Internal rotation and abduction
b. External rotation and abduction
c. Internal rotation and adduction
d. External rotation and adduction

‫نهايه اورثو مرعي‬


224/A patient with neck pain is being examined by a physiotherapist. To test the
semispinalis capitis muscle,which of the following movements is not required ?
a-flexion
b-extention
c-rotation
d-lateral flexion

225/If the therapist is evaluating a patient who has been diagnosed as having hammer toes
and low metatarsal arch deformity,which of the following physical findings could the
therapist detect during the physical evaluation ?
a-metatarsophalangeal joint flexion-proximal interphalangeal joint extension
b- metatarsophalangeal joint hyperextension- proximal interphalangeal joint hyperextension
c- metatarsophalangeal joint abducted-proximal interphalangeal joint flexion
d- metatarsophalangeal joint hyperextension-proximal interphalangeal joint flexion.

226/the therapist is stretching the piceps muscle in a patient with a fracture at the lower
end of the hummers with sever limitation of the joint range of motion.the therapist needs to
perform a very gradual stretch to:-
a- avoid disuse atrophy.
b-avoid hypertrophy.
c-prevent muscle and tendon tear.
d-stimulate muscle.

227/which of the following training exercises can be defined as a training program that uses
selected exercises or activities performed in a sequence?
a-closed chain exercise
b-open chain exercise
c-circuit training
d-isometric exercise

228/A 24year old patient with a shoulder dislocation for 3 weeks,has been referred to a
physical therapist for early mobilization of shoulder joint.the goal behind early mobilization
is:-
a-to prevent stiffness of the shoulder joint
b -to prevent osteoarthritis as a complication of the dislocation
c -to prevent axillary nerve injury
d -to prevent the complication of recurrence of dislocation

229/weakness of which of the following muscles interferes stair climbing,walking up an


incline ,and geeting up from the reclining position?
a-soleus
b-hamstring
c-quadriceps
d-iliopsoas
230/ long jumping athlete female 23 years , comes to outpatient clinic with complain of
instability after touching ground after jump she also says that no previous trauma to knee or
hip what is the cause ?
a – proprioception

231/stretch biceps muscle ?


a - extention shoulder , extention elbow , and pronation

232/what is the most common site of stress (march) fracture ?


a- first metatarsal
b- second metatarsal
c- third metatarsal
d- talus

233/supraspinatus muscle palpation?


a- slight abduction with internal rotation
b- abduction with external rotation

234/patient with ankle sprain injury have pain and red color after take analgesics and rest
the pain subsides which grade of ankle sprain ?
a -1
b-2
c-3
d-4

235/postoperative arthroplasty complication?


a- fracture
b- dislocation
c- infection

236/which fracture is treated by traction ?


a - femur fracture
b- ankle fracture
c-knee fracture

237/patient can't make hyperextension


of shoulder and put his hand in his pocket according to CIF?
a- activity limitation
b- impairment
c- disability

238/partial painful arch of ROM between 90-120 what is the cause ?


a- supraspinatus impingement
b-adhesive capsulitis

239/pt put patient in prone and knee flexion and start make compression of knee joint
against plinth and make twisting movement lateral and medial to test ?
a-Ligament tear
b- Meniscal tear

240/patient have click of knee during going down stairs:-


a- hamstring weakness
b- tibialis anterior weakness
c- rectus femori

241/which muscle make hip abduction + flexion + internal rotation ?


a-Tensor facia latea

242/positive thomas test indicate ?


a-Tight iliopsoas

243/anterior hip dislocation cause ?


a- angle between neck of femur and epicondyle is wide

244/ in shoulder joint the part that increase sliding movement and ROM. ?
a – labrum

245/ spondylolythesis definition ?


a-Vertebra above vertebra move forward

246/ burst fracture ?


a-Humerus
b-Vertebra
c- Ankle
d-spine

245/patient make exercise feel pain in leg and thighs after 48 hrs of exercise ?
a- Delayed onset muscle soreness

266/Contraction in which no change in muscle length:-


a- Isometric

267/type of stretch which is progressive to end of range?


a-ballistic
b- dynamic

268/Palpation for piriforms


a-From ASIS to greater trochanter
b-From PSIS to greater trochanter
c-From coocyx and iliac creast
d-From sacrum to greater trochanter

269/patient have problems in ROM and muscle weakness is classified as


a-participation limitation
b-functional impairment
c-personal factors
d-activity limitation
270/Main purpose from semi lying position:-
a-Gravity assistance
b-Efficiency

271/Before strength hip flexors we do:-


a-Knee extension
b-Extension hip
c- Internal rotation hip

272/Stretch for wrist flexor repair in


a-2 days
b-3 days
c-4 days
d-5 days ** coz patient immobilized in first 3-5 days after surgery

273/Which intensity is most appropriate for normal elder adult for ex


a-Low intensity
b-Moderate intensity
c-High intensity
d- Extreme

274/Patient have 4/5 muscle power what is the minimum number of sessions per week to
achieve full strength ?
a-3
b-4
c-5

275/Exercise in elderly to keep bone density ?


a-Aerobic
b-Resistive
c-Hydro exe

276/Patient have locking knee during going down stairs?


a-Hamstring weaknes
b-Rectus femoris

278/exercise professional autonomy


a-Hard work
b-Patient safety
c- Critical thinking
d- All of the above

279/Benefits from ex?


a-Larger lung volume
b- Normal tidal volume
c- Larger vital capacity
d-Maximum minute ventilation
280/To do max strech to biceps
a-Ext shoulder, ext elbow pronation
b- Ext shoulder, ext elbow supination
c- Lat rot shoulder ext elbow pronation
d-Lat rot shoulder , ext elbow supination

281/football player fall in full knee flexion and medial rotation injury to
a- Acl
b-Pcl ** if there is meniscus we choose it first
c-Medial collateral

282/the most common action cause acl injury


a- Lateral twist
b- Posterior blow to knee
c- Anterior blow to knee

283/convex concave role of knee joint in open chain movement


a-Tibia roll and glide on femur in the same direction
b-Tibia roll and glide on femur in different direction **choose it in closed chain
c-Femur roll and glide on tibia in same direction
d-Femur roll and glide on tibia in different direction

284/Guidelines for safe stretching which of the following ?


a- Slow low force
b-straight line

285/ Regarding closed chain exercise, what of the following?


a-No resistance to disal part
b- avoid strengthening from 45 to 15 knee extension
c- avoid strengthening 90 120 flexion -
d-Knees doesn't go posterior to toes

286/30 years old man. Shoulder pain near deltoid insertion up to shoulder cap refers down
lat arm to elbow. Which is the reason for the pain
A. Teres minor
B. Levator scapula
C. Deltoid
D. Coracobrachialis

287/ which fiber support pelvic floor


a- ant fiber
b- post fiber
c- antro post d circle

288/Which movement not accompany with hyperextension of cervical spine


A-poking chin
B- decrease flexion of lower cervical spine
C-increase activity of scm and levator scapulae
289/Burst fracture, which area affected?
a- Spine
b- talus
c- scaphoid

290/Which negative factor you treat if you increase strength of ms and rom a-a-Personal
factor
b-Activity limitation
c- Participant
d- Impairment

291/Which ms using in climb stairs , go up inclin:-


a- Hamstring
b-Soules
c-Quadriceps
d- Ankle planer flexor

292/Which of the following has the greatest percentage composition of type 1 slow twitch
fibers:-
A-gastronomies
B-supraspinatous
C-biceps brachii
D-biceps femoris

293/Which of the following muscles has least effect in Q angle :


A- lateral retinaculum
B- biceps femoris (second most common)
C- quad
D- iliosoas
E- tibials ant

294/During weight bearing a soft tissue contracture resulting in supination of the forefoot
will be compensated by:-
A- pronation of forefoot
B- pronation of rear foot
C-supination of rear foot
D- pronation of rear and fore foot

295/To decrease load on medial aspect of knee?


A- lateral heel wedge
B- soft knee splint
C- strength extensor

296/Football playrr fall in full flexion and media rotation ?


A-Acl
B-Pcl (second most common choose it if there is no meniscus)
C-Medial collateral
D-medial meniscus
161- isometric contraction as a test in minor lesion:-
A-Strong painful
B- Strong painless
C-Weak painful
D- Weak painless

162/Passive stretch in healing stage:-


A- Acute
B-Sub acute
C-Chronic

163/Osteomyolitis abcess spread


A-Acute stage
B-Sub acute stage
C- Chronic stage

164/Patient has lateral ankle minor pain at rest, palpation increase pain ?
A-Grade 1
B- grade 2
C- grade 3

165/ Guidelines for safe stretching which of the following ?


A-Slow
B-Low force
C-straight line

166/Stages of spondylolisthesis, 75 % displacement ?


A-1
B- 2
C- 3
D- 4

167/PNF which include repetive contractions of the antagonist pattern ?


ARevesal hold
B- repetitive contractions
C-rythmic initiation
D- rythmic stabilization

168/PNF which include after relaxation, passive, active assisted , and resisted contractions of
the agonist pattern ?
A-Revisal hold
B- repetitive contractions
C- rhythmic initiation
D- rhythmic stabilization

169/ Treatment of myositis ossificans?


A-Active and active assisted movement ( avoid stretching)
B- massage
C- strengthening ex
D- stretching ex

170/After strengthening exercises for elderly, what should done to prevent OA in elderly ?
A-Stretching
B- non wt bearing
C- flexibility
D- resistive training

171/ What is the most appropriate preventive intervention in elderly?


A-Medical review
B- balance ex
C- individualized program
D- strengthening ex.

172/which muscle do the first 30 degree of abduction:-


A-supraspinatus

173/21 year old female patient presents with neck pain and stiffness that has gradually
worsened over the last two weeks. Upon examination, the patient is noted to have pain with
left side bending with left rotation and reports pain on the left at the C5-6 junction.
Hypo mobility is also noted with right side-gliding of C6. Which of the following techniques
will be most appropriate to decrease pain?
A- Closing technique for the mid- thoracic spine.
B- Closing manipulation in extension for C5-C6
C- Gapping manipulation in flexion for C5-C6.
D-Flexion/opening manipulation for mid-thoracic spine

174/pt complain of groin pain with limited abduction with external rotation you expect the
diagnosis is :-
a-inguinal hernia
b- adductor strain
c- abductor sprain
d- external obliques weakness

175/Patient have lumbar lordosis when he stand and disappears when he sits we stretch:-
A-Iliopsoas
B-Gluteus maximus
C-Flex lumbar

176/ patient walking on slippery surface we teach him to do:


A-COG keeping close to the body
B-Wide base of support
C- Walking fast

177/muscle waste recovery to normal after?


A-2-3 months (10 weeks )

178/ the best hand placement to palpate piriforms : From ASIS to greater trochanter:-
A-From PSIS to greater trochanter
B-From coccyx to iliac crest
C-From sacrum to greater trochanter

179/semi fowler position?


A-More accesses
B-Gravity assisting position

180/guyon tunnel Injury of which nerve:-


a-ulnar
b-radial
c-median
d-peroneal

181/what should do to prevent Osteoarthritis in elderly:


a-stretching ex
b-strengthening ex
c-flexibility ex
d-non weight bearing ex

182/what is the most appropriate intervention to prevent falling in elderly:


a-medical review
b-balance ex
c- individualized program
d-strengthening ex

183/Most common cause of ACL injury ?


1 - Hyperextension of knee
2- lateral twist
3-posterior blow to knee
4- anterior blow to knee

184/What decrease symptoms of spinal stenosis :


a- 0 flexion
b-0 extension
c-15 extension
d-15 flexion

185/ patient has scoliosis c on rt thoracolumbar we found:-


a-lat flexion of thoracolumbar on rt convex
b-lat flexion and lt convex
c-lat flex and concave on lt side

186/ which fiber support pelvic floor:-


a- ant fiber
b-post fiber
c -antro post
d-circle

187/passive stretch used in healing stage:


A- acute
B -subacute
C- chronic
188/During extreme exercise in a well conditioned athlete blood flow through skeletal
muscle can increase to :
A- 15-25 fold rising to 50to 80 ml/min/100g of ms
B- 6-7ml/min/100g of ms
C- averages 1-2ml/min/100g of ms
D- it’s not estimated

189/ What does mean by partial osteotomy :


A-internal fixation of bone
B-bone sectioning
C-remove periosteum
D-epiphysis fixation

190/ Fracture of Lowe third of humerus cause stiffness in elbow physiotherapist must do
very gentle stretch to avoid:-
A- disuse muscle atrophy
B-hypertrophy
C-muscle and tendon tear ( also to avoid myositis ossificans)

191/ Patient Feel a pain in insertion of this ms. When abduct shoulder and last degree of
abd:-
A-frozen
B- rottator cuff tear
C-supraspinatus tendinitis
D- AC tear

192/In carpal tunnel syndrome the most affected ms is:


A-flexer digitorum profundus
B-flexor carpi ulnaris
C-adductor pollicis longus

193/Small angle between neck and shaft of femur accompanying with:-


A-genu varus
B-genu valgum
C-recrvatum knee

194/Patient with total hip replacement and in acute stage we do:-


A-daily
B-three to four times a weak
C-twice a weak

195/Patient can't do abduction and external rotation due to :


A-dductor sprain
B-adductor strain
C-external oblique strain

196/Which of the following didn't pass through acromiohumeral?


A-supraspinatus
B-tendon of long head of biceps
C-infraspinatus
D-superior portion of glenohumeral capsule

197/Sacral dimple at what level?


A-S1
B-S2
C-S3

198/ Infection of microorganism lead to:


A-osteomyelitis
B-Rocket
C-Equenovarus

199/housemaid knee is?


A- suprapatellar bursitis

200/ athlete felt pain in thigh 48h after running?


A-Synovitis
B- Cramp
C- Delayed muscle sureness

201/pateint has accident brachial plexus injury and comminuted fracture elbow
recommended to total elbow replacement . what contraindication of this surgery?
A-Not enough bone stock
B-Paralysis of forearm flexors
C-Myositis ossificans

202/ to benefit from muscle strength, power and performance, muscles must have?
A-endurance

203/back pain radiated to L.L increase with climbing, lying and sitting?
A-Lumbar
B-Sacroiliac

204/Site of pain of planter fasciitis:-


A- Anterior to calcareous

205/Patient with shoulder pain during evaluation there is no passive or active ROM in
abduction. And lateral rotation:-
a- Frozen shoulder ( adhesive capsulitis)
b- Supra spinatus tendenitis
c- Rotator cuff tears
d- Acromioclavicular dysfunction
Physiotherapy for chondromalacia
Strengthening exercises for the quads mainly vastus lateralis, tapping of patella and mobilization << lateral gliding

206/In treatment of chondromalacia which not needed?


A- Strengthening exercises for the quads
B- Strength hamstring
C- Ice application

207/An abnormal Babinski reflex indicates damage to the:-


a-Spinal cord
b- Brainstem
c-cerebellum
d- Pyramids

208/While assessing the standing posture of a patient, the therapist notes that a spinous
process in the thoracic region is shifted laterally. The therapist estimates that T2 is the
involved vertebra because he or she notes that it is at the approximate level of the:-
a- inferior angle of the scapula
b- superior angle of the scapula
c- spine of the scapula
d-xiphoid process of the sternum

209/you have been treating a 61 year-old women over a period of four months for
adhesive capsulitis which had resulted I a "frozen shoulder". Her recovery has been
good,however she still complains that she cannot reach the upper shelves in her kitchen
cupboards and closets. To help her achieve his goal, joint mobilization should focus on:-
a-inferior glide
b-posterior glide
c-anterior glide
d-grade II oscillations

210/three months following a traumatic injury to the hand,which had resulted in surgical
tendon repair and fracture stabilization,a physical therapist is planning a treatment
program to address tightness of the lumbricals. Exercises to increase ROM will be best
utilized if:-
a-both the MCP and IP joints are moved into flextion
b- both the MCP and IP joints are moved into extension
c-the MCP joints are extended and the IP joints are flexed
d- the MCP joints flexed and the IP joints are extended

211/your interview with an 18year-old female cross country runner elicits a history of
stiffness and diffuse ache in her right knee which is aggravated by prolonged sitting.
Based on this information, your physical examination should focus on the likely diagnosis
of:-
a-iliotibial band friction syndrome
b-Osgood-schlatter disease
c-meniscal tear
d-patellofemoral syndrome
212/ Following cast immobilization for a now healed supracondylar fracture of the humerus,
a patient’s elbow lacks mobility. To increase elbow range of motion, joint mobilization
in the maximum loose-packed position should be performed at:
A- full extension
B- 90 degrees of flexion
C- 70 degrees of flexion
D-30 degrees of flexion

213/ A patient with degenerative joint disease of the right hip complains of pain in the
anterior hip and groin, which is aggravated by weightbearing. There is decreased range
of motion and capsular mobility. Right gluteus medius weakness is evident during
ambulation and there is decreased tolerance of functional activities including transfers
and lower extremity dressing. In this case, a capsular pattern of joint motion should be
evident by restriction of hip:
A-flexion, abduction and internal rotation
B- flexion, adduction and internal rotation
C- extension, abduction and external rotation
D- flexion, abduction and external rotation

214/A patient diagnosed with cervical stenosis coupled with a positive Lhermitte`s
sign,as elicited by the maneuver in the picture, would most likely indicate:-
a-upper motor neuron findings in the lower extremities
b- lower motor neuron findings in the upper extremities
c- lower motor neuron findings in the lower extremities
d-both upper and lower motor neuron finding in the lower extremities

215/damage as a result of Salter-Harris type IV supracondylar humeral epiphyseal


fracture in a young athlete will most likely result in:-
a-refracture at a future time
b-nonunion
c-arrested growth
d-severing of the radial nerve

216/a weightlifter with hypertrophy of the scalene muscle complains of pain paresthesia
in the right upper extremity when lifting the weight overhead. It is most likely that this is
a manifestation of:-
a-thoracic outlet syndrome
b-vertebral artery obstruction
c-cervical radiculitis
d-complex regional pain syndrome type I

217/while crossing the finish line of a 100meter race an athlete lunged and fell on her
dorsiflexed wrist. She immediately complained of pain and tenderness over the radial
aspect of the wrist in the area of the anatomical snuffbox. You suspect a possible
fracture of the:-
a-lunate
b-capitate
c-scaphoid
d-hook of the humate
218/to prevent maximal compressive forces being placed on the patella, a therapist
should minimize placing the patient:-
a- prone and flexion the knee to 30 degrees
b-in a sitting position with the knee flexed to 90 degrees
c-supine and flexing both the hip and knee to 110 degrees
d-prone and flexing the knee to 110 with the hip extended

219/as a result of diminished movement associated with Parkinson's disease a physical


therapist might employ rhythmic initiation primarily to help improve:-
a-trunk stability and proximal tone
b- trunk rotation
c-upper extremity function
d-active and passive ROM

220/a therapist has decided to use mechanical lumbar traction on a patient with
posterior herniated nucleus pulposus 1.2-3. if tolerated by the patient ,the best
positioning for this treatment would be:-
a-prone, with no pillow under the hips or abdomen
b-prone, with a pillow under the hips and abdomen
c-supine, with the hips and knees flexed to 45 degrees
d-supine with hips and knees flexed to 90 degrees

221/A patient has limited motion in supination and calcaneal inversion at the subtalar
joint, using manual techniques, the accessory motion of the calcaneus that needs to be
emphasized in order to increase the motions that are limited would be:-
a-anterior glide
b- posterior glide
c- medial glide
d- lateral glide

222/during weight bearing a soft tissue contracture resulting in supination of the


forefoot will be compensated for by:-
a- pronation of the forefoot
b- pronation of the rear foot
c-supination of the rear foot
d- pronation of the forefoot and rear foot

232/A PT is manual muscle testing the supraspinatus muscle of the shoulder. Which
position best isolates this muscle to test strength?
a- arm neutral, elbow flexed 90 degree, resist external rotation.
b- arm neutral, elbow flexed 90 degree, resist internal rotation.
c-arm abducted 90 degree, horizontally adducted 30 degree,internally rotated,resist
shoulder flexion .
d- arm abducted 90 degree, horizontally adducted 30 degree,externally rotated,resist
shoulder flexion .

233/A PT is performing a positional release technique to the cervical spine. Her goal in
treatment is to open the affected foramen to decrease nerve root irritation. Which
position is best for that goal? 233/B:- T maximally open the
a- forward flexion, contralateral sidebending, ipsilateral rotation. foramen, the neck should be forward
b- forward flexion, contralateral sidebending, contralateral rotation. flexed, contralaterally sidebent and
c- forward flexion, ipsilateral sidebending, ipsilateral rotation. contralaterally rotated. Option A
would be best opining of the affected
d- forward flexion, ipsilateral sidebending, contralateral rotation.
facet joint .

234/A PT is evaluating a patient with a know endocrine dysfunction. During muscle


testing which joint would be expected to show a painless weakness?
234/A:- endocrine system disease
a-hip can be manifested with proximal,
b- elbow painless muscle weakness. Of the
options listed, the hip is the most
c- ankle
reasonable expected weakness.
d- wrist

235/a college student presents with pain in the shoulder that began the week after
finals, but she cannot recall any mechanism of injury. Upon inspection, the examiner
notes scapular winging on the dominant side. During evaluation the patient has difficulty
with elevation greater than 120 degree. She has normal scapular elevation. What would
be the best differential diagnosis ? 235/C:- The long thoracic nerve innervates the serratus anterior. A
a- acute rotator cuff tendonitis scapular stabilizer. The nerve can be compressed with a backpack
b- shoulder impingement strap as it lies along the chest wall. This patient will have scapular
c-long thoracic nerve lesion winging but good scapular elevation strength. Damage to the spinal
accessory nerve will inhibit elevation of the scapula.
d-accessory nerve lesion

236/which of the following exercise schematics uses sets of repetitions at 100 percent,
75 percent, and 50 percent of the ten-repetition maximum, in that order for
strengthening ?
a- Oxford 236/A:- The Oxford technique uses the ten-repetition maximum (10-RM)
b-DeLorme and the protocol is to perform ten repetitions each at 100 percent, 75
percent, and50 percent of the 10-RM with a brief rest between sets.
c- DAPRE
D-DOMS

237/which one of the following has the greatest percentage composition of type I slow-
twitch muscle fibers?
a-gastrocnemius 237/D:- the supraspinatus is a phasic or postural muscle, which indicates it has
higher composition of slow-twitch oxidative muscle fibers. It responds best to
b- biceps femoris
slow-weight, high-repetition exercise.
c- biceps brachialis
d- supraspinatus

238/which grade of joint mobilization would be most appropriate for improving joint
motion using a large-amplitude movement into end range?
238/C:- Grade III Joint mobilizations utilize a large-amplitude mobilization into the
end range of accessory joint motion to improve range of motion .
a- Grade .i
b- Grade .ii
c- Grade .iii
d- Grade .iv

239/A PT is treating a patient with left-sided C4 facet impingement. She plans to use manual
cervical traction to attempt to open the left facet. Which position of the cervical spine would
maximally open the left facet?
a- flexion, left side-bending, left rotation. 239/C:- The motions to maximally open the facet joints are
b- flexion, left side-bending, right rotation. flexion, contralateral side-bending and ipsilateral rotation.
c- flexion, right side-bending, left rotation.
d- flexion, right side-bending, right rotation.

240/ physical therapist is evaluating elbow flexion from neutral position ; which muscle
should be evaluated ?
a- biceps brachii
b- brachialis
c- brachioradialis
d- anconeous

241/ which of the following muscles compress the checks :-


a- Buccinator
b- corregator
c- frontalis
d- Depressor angulii

242/ Which muscle draw eyebrow to upward making horizontal wrinkling:-


a-frontalis ( occiptofrontalis)
b-corregator
c-orbicularis oris
d-lateral ptyroid
243/ muscle that moves angle of mouth inferior and back
A- Platysma
B -Mentalis
c- Masseter

244/ which of the following Closes the jaw:


a- masseter
b- levator lipii
c- mentalis
d- orbicularis oris

245/ which muscle Raise the skin of the chin


and the lower lip:-
a- buccinators
b- levator libii
c- mentalis
d - platysma

246/ which muscle Close and protrude the lips, as in whistling:


a- masseter
b- mentalis
c- orbicularis oris
d- depressor libii

247/which muscle close the eyes?


A)buccinators
B)orbicularis oris
C)orbicularis occuli
D)masseter

248/ Which of the following Draws the eyebrow downward and inward, with vertical
wrinkles:
a- buccinators
b- corrugators
c- frontalis
d- depressor angulii

249/ which of following muscle doesn't closes the mouth:


a- masseter
b- ptrygoidus medialis
c- temporalis
d- suprahoid ms

250/The muscle used for smiling:-


A- Placerus
B- Zygomaticus
C- Platysma
D- Sonreirus

251/ The circular band of muscles that surround the eyes are the ______ oculi, while the
circle muscle that surrounds the mouth is the _____oris. (the same answer for both blanks).
A- Orbicularis
B- Annualus
C- Circularis
D- Muscularis

Muscle Abbr Action


Corrugator COR pulls eyebrows together downward and medially
depressor anguli oris DAO pulls the corner of the mouth downward as in
frowning
dilator naris DIN flares nostrils
depressor labii inferioris DLI draws lower lip downward and laterally
Frontalis FRO raises eyebrows
levator anguli oris LAO elevates the angle of the mouth
levator labii superioris alaeque nasi LLA raises upper lip and dilates nostril
levator labii superioris LLS raises upper lip
Mentalis ME closes and protrudes the lips
N
orbicularis oris inferioris OOI closes and protrudes the lips
orbicularis oris superioris OOS closes and protrudes the lips
orbicularis oculi inferioris OCI closes the eye
orbicularis oris superioris OCS closes the eye
Platysma PLA draws corners of the mouth down, wrinkles neck
Procerus PRO depresses medial corners of eyebrows (frown),
wrinkles forehead
Risorius RIS draws corner of mouth outward as in smiling
zygomaticus major ZYJ raises and draws corner of the mouth laterally
(smile)
zygomaticus minor ZYN raises and everts upper lip

Resting blood flow of skeletal muscle


During rest 1-4 ml/min per 100g Maximal blood flows may reach 50-100 ml/min per 100g
Depending upon the muscle type. Therefore, blood flow can increase 20 to 50-fold with
maximal vasodilatation

255/What is the blood flow at resting?


A-1 to 4 ml/min/110g
B- 6 to7 ml/min/110g
C- 60 to 80 ml/min/110g

*an athlete performing maximum effort, the amount of blood reaching skeletal muscle Or
256/ what is the blood flow for an athlete at max activate?
A- 1 to 2 ml/min/110g
B- 6 to7 ml/min/110g
C- 60 to 80 ml/min/110g
257/Which of these don't pass under the acromion glenoid cavity:-
A-infraspinatus tendon
B-teres major
C-supraspinatus tendon
D-teres minor

258/A physical therapist is beginning an evaluation of a patient with a diagnosis of “knee


strain”. Range of motion limitation does not follow the normal capsular pattern of the knee.
Which of the following are possible
causes of the restriction in range of motion?
a-Ligamentous adhesions
b-Internal derangement
c-Extra-articular lesions
d-All of the above

259/A child presents to physical therapy with a diagnosis of right Sever’s disease. What joint
should be the focus of the therapist’s evaluation?
a-Right knee joint
b-Right hip joint
c-Right wrist joint
d-Right ankle joint

260/A physical therapist is treating a 24-year-old woman with


lumbar muscle spasm. Part of the patient's treatment involves
receiving instruction on correct sleeping positions.Which of the
following would be most comfortable?
A-Supine with no pillows under the head or extremities
B-Prone with a pillow under the head only.
C-Sidelying with a pillow between flexed knees

261/The therapist is treating a 52-year-old woman after right total hip replacement. The
patient complains of being selfconscious
about a limp. She carries a heavy briefcase to and from work every day. The therapist notes
a Trendelenburg gait during ambulation on level surfaces. What advice can the therapist give
the patient to minimize gait deviation?
a-Carry the briefcase in the right hand
b-Carry the briefcase in the left hand
c-The patient should not carry a briefcase at all.
d-It does not matter in which hand the briefcase is carried.

262/While gait training a patient following a cerebral vascular accident, you observe the
knee on the affected side going into recurvatum during stance phase. The most likely cause
of this deviation can be attributed to:
A-severe spasticity of the hamstrings or weakness of the gastrocnemius-soleus.
B-weakness or severe spasticity of the quadriceps
C-weakness of the gastrocnemius-soleus or spasticity of the pretibial muscles
D-weakness of both the gastrocnemius-soleus and pretibial muscles

263/Pt has calf msc tear type of massage used


A.taptman
B.vibration
C.efflurag
D.deep friction

264/A physician has ordered a physical therapist to treat a patient with chronic low back
pain. The order is to “increase gluteal muscle function by decreasing trigger points in the
quadratus lumborum”. What is the first technique
that should be used by the physical therapist?
A-Isometric gluteal strengthening
B-Posture program
C-Soft tissue massage

265/in acute of lymphedema:-


a-Begin with proximal part then distal
b-Begin with distal part then proximal
c-Superficial efflurage
Ms Test Questions

266/ which method used in the measurement of shoulder abduction:


a-tapemeasurment
b-goniometer
c-visual analogue scale

267/The therapist is assessing a patient’s strength in the right shoulder. The patient has 0o
of active shoulder. abduction in the standing position. In the supine position, the patient has
42o of active shoulder abduction and 175o pain-free passive shoulder abduction. What is
the correct manual muscle testing grade for the patient’s shoulder abduction?
a-3-/5 (fair -) 267/C- 2-/5 (poor ) , poor grade The ability to move through a partial arc ofmotion in the
b-2+/5 (poor+) horizontal plane is graded as poor - The grade of poor means that the muscle is capable of
c-2-/5 (poor-) completing the range of motion in the horizontal plane.
d-1/5 (trace) The grade of poor+ denotes the ability to move in the horizontal plane to completion of the
range of motion against resistance or to hold the completed position against pressure. It
also means that the muscle is capable of moving through a partial arc of motion in the
antigravity position.

c. Because the patient does not have 50% of normal range of motion
in the gravity eliminated position, 2-/5 is the appropriate grade Some therapists argue
that this is an example of a 1 + /5 grade Sources used in preparation of this exam indicate
that there is no grade of 1 +/5 with manual muscle testing.

268/While evaluating a patient who suffered a complete spinal cord lesion, the therapist
notes the following strength grades with manual muscle testing: wrist extensors = 3+/5,
elbow extensors = 2+/5, and intrinsic muscles of the hand
= 0/5. What is the highest possible level of this lesion?
A- C3
B- C4
C-C7

269/A and C are equally correct A patient is referred to physical therapy with a history of
temporomandibular joint pain. The therapist notices that the patient is having difficulty
closing his mouth against minimal resistance.With this information, which of the following
muscles would not be a target for strengthening exercise to correctthis deficit?
A-Medial pterygoid muscle
B-Temporalis
C-Masseter
D-Lateral pterygoid muscle

270/A patient is in prone position with his head rotated to the left side. The left upper
extremity is placed at his side and fully internally rotated. The left shoulder is then shrugged
toward the chin. The therapist then grasps the mid shaft of the patient’s left forearm. The
patient is then instructed to “try to reach your feet using just your left arm.” This movement
is resisted by the therapist. The test is assessing the strength of what muscle?
A- Upper trapezius
B- Posterior deltoid
C-Latissimus dorsi
D- Triceps brachii

271/patient is positioned in the supine position. The involved left upper extremity is
positioned by the therapist in 90o of shoulder flexion. The therapist applies resistance into
shoulder flexion, then extension. No movement takes place. The therapist instructs the
patient to “hold” when resistance is applied in both directions. Which of the following
proprioceptive neuromuscular facilitation techniques is being used?
a-Repeated contractions
B- Hold-relax
C -Rhythmic stabilization
D-Contract-relax
C- Rhythmic stabilization
Hold-relax (HR)-autogenic inhibition: a relaxation technique usually performed at the point of limited range of motion in the agonist pattern
An isometric contraction of the range-limiting antagonist is performed against slowly increasing resistance
This is followed by a voluntaly relaxation by the patient and then passive movement of the extremity by the clinician into
Hold-relax-active (HRA)-reciprocal inhibition: following application of the hold-relax technique, the .the newly gained range of the agonist pattern
patient performs an active contraction into the newly gained range of the agonist pattern Contract-relax (C-R): a relaxation technique usually
performed at the point of limited range of motion in the agonist pattern An isotonic movement in rotation is performed followed by an
isometric hold of the range-limiting muscles in the antagonist pattern against slowly increasing resistance, voluntary relaxation, and active movement
into the new range of the agonist pattern

272/A physician prescribes isotonic exercises for the left biceps brachii. Which of the
following exercises is in compliance with this order?
A-Biceps curls with the patient actively and independently flexing the left elbow using a 5-
pound dumbbell as resistance
B-Rhythmic stabilization for the left elbow.
C-Elbow flexion at 90o per second with speed controlled by a work stimulator.
d-None of the above.

273/The therapist is performing an orthopedic test on a 25-year-old man with the chief
complaint of low back pain. The patient has a positive Thomas test. With this information,
what might the therapist need to include in the treatment plan?
A- Stretching of the hip abductors 272/D- stretching of the hip flexors because Thomas test make for hip
b- Stretching of the hip adductors Flexion.
c- Stretching of the hip extensors The Thomas test is a screen to determine whether the hip flexors
d- Stretching of the hip flexors are too tight.
274/stretching for which muscle:-
a)lateral rotator
b)adductor
c)hamstring
d)calf muscle

275/ During an evaluation, the therapist taps on the flexor retinaculum of the patient’s
wrist, which causes tingling in the thumb. What test is this? For what condition does it
screen?
A-Phalen’s test – carpal tunnel
B-Finkelstein test – de Quervain’s disease
C-Tinel’s sign – de Quervain’s disease
D-Tinel’s sign – carpal tunnel

275/D- A positive Tinel 's sign screens for carpal tunnel syndrome when the tapping force is performed
over the carpal tunnel itself. In Phalen's test, the therapist places the patient's wrists in maximal
flexion and holds for 1 minute. The test is positive if there is paresthesia in the median nerve
distribution. The Finkelstein test screens for de Quervain's disease by allowing the patient to make a
fist with the thumb wrapped in the fingers. The test is positive if there is pain over the adbuctor pollicis
longus and extensor pollicis brevis tendons.

276/A 27-year-old woman is referred to a physical therapy clinic with a diagnosis of


torticollis. The right sternocleidomastoid is involved. What is the most likely position of
the patient’s cervical spine?
a-Right lateral cervical flexion and left cervical rotation 276/a. Torticollis involving the right
b-Right cervical rotation and right lateral cervical flexion sternocleidomastoid would cause right lateral
c-Left cervical rotation and left lateral cervical flexion cervical flexion and left cervical rotation.
d-Left lateral cervical flexion and right cervical rotation.

277/A therapist assessing radial deviation range of motion at the wrist


The correct position of the goniometer should be as follows: the proximal arm aligned with
the forearm and the distal arm aligned with the third metacarpal. What should be used as
the axis point?
a- Lunate
b- Scaphoid
c- Capitate
d- Triquetrum

278/A therapist is instructing a patient in the use of a wrist-drive prehension orthotic. What
must be done to achieve opening of the involved hand?
A-Actively extend the wrist This type of orthotic uses tenodesis to achieve
B-Passively flex the wrist opening and closing of the hand. To close the hand,
C-Actively flex the wrist the patient actively extends the wrist. To open the
D-Passively flex the wrist hand, the patient passively flexes the wrist.

279/long measurement of lower limb from :


a- ASIS to medial malleulus
b- ASIS to lateral malleulus
c- greater trochanter to lat malleulus

280/The most appropriate position to strength sternocleidomastoid is:-


a- supine
b- sitting
c- prone
d- standing

281/which is not correct about oxford scale?


a- 0 no contraction
b- 1 flicker contraction
c- 4 can make full ROM against resistance

282/when evaluation of hip abduction of lt hip, it was poor grade, so:-


a- can apply full ROM of abduction in side lying
b- can apply full ROM of abduction in supine
c- can apply half ROM of abduction in supine
d- can apply full ROM of abduction with

283/when you examine a muscle with no evidence of contraction, which muscle grade
would you give:-
a- Poor
b- Trace
c- Good
d- zero

284/Contration and lastly resistive movements it’s


A- Rhythmic initiation
B- Rhythmic stabilization
C- Contract-relax
D- Hold-relax

285/A 35-year-old woman with a diagnosis of lumbar strain has a physician’s prescription
with a frequency and duration of 3 sessions/week for 6 weeks. The physical therapy
evaluation reveals radiculopathy into the
L5 dermatome of the right lower extremity, increased radiculopathy with lumbar flexion,
decreased radiculopathy with lumbar extension, poor posture, and hamstring tightness
bilaterally at 60o. What is the best course of treatment?
a-Lumbar traction, hot packs, and ultrasound.
b-McKenzie style lumbar extensions, a posture program, hamstring stretching, and a home
exercise program.
c-McKenzie style lumbar extensions, a posture program, hamstring stretching, home
program, hot packs, and ultrasound.
d-Lumbar traction, hot packs, ultrasound, and hamstring stretching.

285/B:- Hot packs are not indicated because there is no mention of abnormal muscle tone. The entire lumbar area is too much
surface area for ultrasound. An argument could be made for lumbar paired with heating modalities in all of the answers.
286/A 42-yearold receptionist presents to an outpatient physical therapy clinic complaining
of lowback pain. The therapist decides that postural modification needs to be part of the
treatment plan.What is the best position for
the lower extremities while the patient is sitting?
a- 90o of hip flexion, 90o of knee flexion, and 10o of dorsiflexion
b- 60o of hip flexion, 90o of knee flexion, and 0o of dorsiflexion
c- 110o of hip flexion, 80o of knee flexion, and 10o of dorsiflexion
d- 90o of hip flexion, 90o of knee flexion, and 0 of dorsiflexion

287/from supine when there is shortening of this ms the arm rased above table and not
down ,the ms is:-
a-biceps
b-brachioradialis
c-brachialis
d-pectoralis major

288/PT is evaluating patient who is unable to open the door using supination ; so he expects
weakness of:-
a- brachioradialis
b- biceps brachii
c- anconeus
d- flexor carpi ulnaris

289/physical therapist is evaluating elbow flexion from neutral position ; which muscle
should be evaluated ? *Flex Supination_ biceps
a- biceps brachii *Flex Mid position_ brachialis
b- brachialis *Flex prnation_ brachioradialis
c- brachioradialis
D- anconeous 289/C:- flex with supination biceps, flex with neutral position brachioradialis, flex with
pronation brachialis.

290/some tests performs to dect ligament& capsule stability in shoulder joit when
positioned in abduction lateral rotation with continued anterior propagation of humeral
head this due defect in ligaminta in:-
A- superior part
B- inferior part
C- anterior part

291/A patient presents to an outpatient clinic with complaints of shoulder pain. The
therapist observes a painful arc between 70o and 120o of active abduction in the involved
shoulder. This finding is most indicative of what shoulder pathology?
a-Rotator cuff tear
C. The "painful arc" is most indicative of shoulder impingement. The
b-Acromioclavicular joint separation
soft tissues of the shoulder are pinched under the acromion process
c-Impingement
at approximately 60 degrees tol20 degrees of abduction. Pain
d-Labrum tear
throughout abduction active range of motion suggests
acromioclavicular joint dysfunction

292/A 67-year-old woman presents to an outpatient facility with a diagnosis of right


adhesive capsulitis. The therapist plans to focus mostly on gaining abduction range of
motion. In which direction should the therapist mobilize the shoulder to gain abduction
range of motion?
a-Posteriorly 292/C:- The therapist must stretch the inferior portion of the
b-Anteriorly capsule in an effort to gain abduction of the involved shoulder. This
principle is supported by the convex-concave rule.
c-Inferiorly
d-Superiorly

293/A therapist places a pen in front of a patient and asks him to pick it up and hold it as
he normally would to write. The patient picks the pen up and holds it between the pad of
the thumb and the middle and index fingers. Why type of grasp or prehension is the
patient using?
A-Palmar prehension
B-Fingertip prehension
C-Lateral prehension
D-Hook grasp

294/ selfactuallization mean:-


a-person frustrated because cannot gain target
b- sense of blessing due to age
c-person has gain something and proud
d-no sense

295/A 17 year-old gymnast has been diagnosed with an unstable spondylolisthesis


involving L5 on S1. Appropriate management of this problem by a physical therapist
should include:
A-back extension exercises.
B-abdominal strengthening and flexion exercises.
C-joint mobilization emphasizing lumbar rotation in both supine and sideling.
D-lumbar extension mobilization using hand pressure directed anteriorly.

296/ patient with posterior lumbar disc herniation by physiotherapist apply mechanical
lumbar traction put patient on:-
a-supine with towel under back
b-supine with flexed knee
c-prone with towel under abdomen
d-prone lying

297/Capsular tightness has limited your patients ability to fully extend her knee.
Treatment to restore joint motion:-
a- anterior glide , external rotation of tibia
b- anterior glide , internal rotation of tibia
c- posterior glide, external rotation of tibia Anterior Glide_ increase Ext.
d- posterior glide, internal rotation of tibia Post Glide_ increase flex.
298/Boy have 16 years old have acute knee sprain after chronic we advice:-
a- knee protection
b- knee immobilization
c- range of motion
d- hot application

299/Following cast immobilization for a now healed supracondylar fracture of the humerus,
a patient’s elbow lacks mobility. To increase elbow range of motion, joint mobilization in the
maximum loose-packed position should be performed at:
A-full extension
B-90 degrees of flexion
C-70 degrees of flexion
D-30 degrees of flexion

300/During under water exercises which will be difficult:-


a- moving the limb horoziantly
b- moving the limb toward bottom ( against bouncy force of water )
c- moving the limb to the surface

301/Which of the following is not benefit in physiotherapy:


a-increase muscle fiber size
b-increase heart rate
c-increase ligaments and tendons
d-increase the power structure of bones

302/inorrect about poor posture:-


A-need surgery and physiotherapy
Not sure as we lost the 4th choice
B-spinal and frozen shoulder problems
C-pain and affect strength of muscles

303/Application of intermittent traction to cervical depend on:


a- wt of patient
b- localization of pain
c- proper diagnosis and knowing problem

304/Which of the following is the normal end-feel perceived by an examiner assessing


wrist flexion?
a-Bone to bone
b-Soft tissue approximation
c-Tissue stretch
d-Empty

305/Aphysical therapist should place the knee in which of the following positions to palpate
the lateral collateral ligament ( LCL)?
a-Knee at 60o of flexion and the hip externally rotated.
b-Knee at 20o of flexion and the hip at neutral.
c-Knee at 90o of flexion and hip externally rotated.
d-Knee at 0o and the hip at neutral

306/A therapist is mobilizing a patient’s right shoulder. The movement taking place at the
joint capsule is not completely to end range. It is large amplitude movement from near the
beginning of available range to near the end of available range. What grade of mobilization
according to Maitland, is being performed?
a-Grade I
Grade I- Small amplitude rhythmic oscillating mobilization in early range of movement
b- Grade II Grade II- Large amplitude rhythmic oscillating mobilization in midrange of movement
c-Grade III Grade III- Large amplitude rhythmic oscillating mobilization to point of limitation in range of
d-Grade IV Movement
Grade IV- Small amplitude rhythmic oscillating mobilization at end range of movement
Grade V- (Thrust Manipulation) - Small amplitude, quick thrust at end range of movement

307/You wish to mobilize a patient’s shoulder using an inferior glide technique. It would be
best to use this technique be propositioning the patient’s arm in:-
a- 95 degrees of abduction with lateral rotation
b- 125 degrees of abduction and internal rotation
c- 55 degrees of abduction and neutral rotation
d- 95 degrees of shoulder flexion and neutral rotation

You might also like